« first day (13 days earlier)      last day (561 days later) » 
00:00 - 16:0016:00 - 00:00

4:00 PM
0
Q: Functional equaliton on continuous function $f:[0,1]\to \mathbb{R}$

Raheem NajibLet $f:[0,1]\to \mathbb{R}$ be continuous, and suppose that $f(0)=f(1).$ Show that there is a value $x\in [0, 1998/1999]$ satisfying $f(x)=f(x+1/1999)$.

Short question.
 
0
Q: Should "biased views and crackpottery" comments be deleted?

OokerFor example, the comments in this answer: Entropy and the principle of least action. As they are viewed as biased views and crackpottery, should we delete them? Should I flag them if I meet?

 
0
Q: Fouriertransform 1/(t^2+6t+10)

ShrubberI want to solve the equation $$\int^{\infty}_{-\infty}\frac{u(s)}{1+4(t-s)^2} ds= \frac{1}{t^2+6t+10}$$ using $$\mathcal{F}[\frac{1}{1+t^2}] = \pi e^{-|w|} $$ The left hand side is a convolution which I can solve using the hint. However I don't know how I Fourier Transform the right side. When...

Short title.
0
Q: Generating function for the set of $w$-free words.

Benjamin XSuppose $X$ is an alphabet and $w \in X^n$ is a word over it. Consider a set $P$ of $w$-free words over $X$ (a word is called $w$-free if it does not contain $w$ as a subword). I want to write a generating function for $P$ (by length). I am trying to write some kind of a recurrence relation consi...

Question contains please.
0
Q: Adding linearly independent row vectors to a matrix.

shzanSuppose we are given a matrix $M_{n*2n}$ of $n$ linearly independent row vectors. Then I am trying to find an algorithmic way to add $n$ more linearly independent row vectors to this matix resulting in to a matrix $M_{2n*2n}$. Consider this easy example, if the given matrix $M_{2*4}$ is \begin{...

Question contains thank you.
0
Q: Water volume sum

Mission CodingLet's start by quoting the question. Water flows at the rate of 10 m per minute through a cylindrical pipe of diameter 5mm. How much time would it take for it to fill a conical vessel of diameter 40 cm and depth 24 cm? I just don't get what the question means. After doing many sums on volu...

Short title.
0
Q: normal distribution, mean=20, sd=2 , p(x^2-x<2) ???

Selami Hoxhanormal distribution, mean=20, sd=2 , p(x^2-x<2) i dont know how to convert it to z-score form. i think if we put 2 on left side we get the p((x-2)(x+1)<0) form. but i dont know how to get any further?

Title contains ??. Short question.
0
Q: Vertex cover problem instance

user290314I apologize for a (probably) trivial question but I am looking for a problem name so i can google for its potential solutions. The problem is: Unlike in minimum vertex cover problem where the goal is to compute a set of vertices covering all the edges in a given graph, I am looking for a version ...

Short title. Title contains problem. Question contains thank you.
-1
Q: HELP ME WITH THIS

wasifQuestion 1 In a car racing game a Ford mustang car sets off from Edinburgh towards London Kings Cross (800km away) at 6am. It travels at an average speed of 300km per hour. After you have progressed some levels with your game, a Renault car starts at 8.00am in the opposite direction, from London ...

Welcome to Math.SE, wasif. Words such as help do not add information to titles. Please edit the title so that it better describes the specifics of your question. Do not hesitate to make it longer or include a formula if needed. More tips here. (autocomment)Normal Human 31 secs ago
0
Q: variation metric tensor

Saladinos(I asked this on physicsexchange but no reply) I have the metric tensor $g_{\mu\nu}$. I want to make the variation of $\sqrt{-g}$ where $g=detg_{\mu\nu}$. Can I make this work? $\sqrt{-g}=\sqrt{-e^{Tr(log(g_{\mu\nu}))}}=\sqrt{-e^{-Tr(log(g^{\mu\nu}))}}\rightarrow\delta\sqrt{-g}=\delta(\sqrt{-e...

Short title.
0
Q: probability to get faster code ...........

user290319i have my code with the aim of speeding it up, and re-run the experiment on the same computer The following times are observed: 606 567 535 551 536 462 535 508 501 492 523 449 598 576 548 i have to submit report so please help me ...... With what significance is the new code faster than the or...

0
Q: How to find the x(t) and y(t) functions in the Lotka Volterra Equations?

Mary AnneI'm doing this math project involving the Lotka Volterra Equations. My goal was to be able to graph both the x(t) and y(t) on the same axis against time. So far I was able to find the combined equation of x' and y' using partial differentiation. I called it E(x,y). Substituting my known constant...

Question contains thank you.
Question contains please.
0
Q: Random variables and reference probability measure

KolminI am self-studying probability theory, and I am having quite some problems with the very basic concepts of the theory that are seriously hampering any attempt to proceed further in my study. Here there is one basic problem I am having, with my thoughts (written in italic) about it. Assume the...

Question contains thank you.
0
Q: Calculate the variation of J defined on C([0,1]) by J(Y)

Guten TagCalculate the variation of J defined on C([0,1]) by J(Y)= integral 0 to 1 (Y(x))^2 + (Y'(x))^2 - 2Y(x)sin(x))dx.

Short question.
0
Q: Help with proof of expected value of gamma-gamma distribution

ahmadmathematically solution for Help with proof of expected value of gamma-gamma distribution gamma-gamma distribution

Title contains help. Short question.
0
Q: idempotents in a subalgebra of $B(H)$.

jokerLet $\mathcal{A}$ be a sub-algebra of $B(H)$ such that $\mathcal{A}$ generated by all its idempotents and $\mathcal{A}$ is close under weak operator topology. Suppose that there exist idempotents $P1$ and $P2$ in $\mathcal{A}$ such that $P1P2≠P2P1$. Can we say there exist idempotent $P3$ in such...

Question contains please, in advance.
0
Q: Finding a formula for a transformation

b.vasquezFind a formula in the form z → uz + v or z → uz + v for the following transformation: the rotation through π/2 about i.

Short question.
0
Q: $\lim_{n\to\infty}\sum_{n=0}^\infty x^n$

John PartFind: $$\lim_{n\to\infty}\sum_{n=0}^\infty x^n$$ Where x is a set of the complex numbers and $|x|<1$ Then prove the limit using $\epsilon$ and $N_0(\epsilon)$ I think the limit is either $0$ or $\frac{1}{1+x}$ but i'm not sure which. Given either I don't know how to do the proof

Welcome to Math.SE, John Part. A title should not be all-MathJax; having some plain text helps with search and navigation. (autocomment)Normal Human 37 secs ago
0
Q: If A and B are bounded sets show that A⊂B implies diam(A)≤ diam(B).

As Mabelow is what I attempt, let diam(A) diam(B) => m>n which contradicts our supposition that A⊂B thus , diam(A)≤ diam(B). is my solution is correct?

Short question.
0
Q: Tangent space of open set

Sahiba AroraIf U is a open subset in R^n and p is a point in U, then tangent space of U at point p is the whole of R^n. I am having difficulty understanding why this is true.

Short title. Short question.
0
Q: LU Factorization of a full rank sqare matrix.

midiIf A is an invertible matrix then a necessary and sufficient condition for the LU Factorization to exist is : If A is invertible, then it admits an LU (or LDU) factorization if and only if all its leading principal minors are nonsingular Source : https://en.wikipedia.org/wiki/LU_decompositi...

Question contains in advance.
 
4:38 PM
0
Q: Does the question count inculde duplicates?

JosephAt the time of writing this there where 64,202 questions according to the figure given on the questions-newest tab. What does this number represent, is it the total number of questions on the website, the total number of unique questions (i.e. so that duplicates aren't included), does it include ...

 
0
Q: Linear equations for set

AlbertLet $U$ be a set. $Sub(U) := \{A | A \subset U \}$. We can trivial define binary operations $\cup$ and $\cap$ on $Sub(U)$. What van we say about linear equations on $(Sub(U), \cup, \cap)$? For example, about linear equations with matrix $2 \times 2$?

Short title.
0
Q: Continuity of Operator Norm

SamLet $L(\mathbb{R}^n, \mathbb{R}^m)$ be the set of linear maps from $\mathbb{R}^n$ to $\mathbb{R}^m$ and $\phi: L(\mathbb{R}^n, \mathbb{R}^m) \to \mathbb{R}$ be defined by $A \mapsto \|A\| := \sup\{\|Ax\| : \|x\| = 1\}$. I'm trying to give a direct proof that $\phi$ is continuous but I'm having ...

Short title.
0
Q: Extended Liouville’s theorem

IDontKnowMathI know that the Extended Liouville’s theorem state that: $An$ $entire$ $function$ $f(z)$ $with$ $|f(z)|<C|z|^n$, $for$ $natural$ $number$ $n$, $is$ $a$ $polynomial$ $of$ $degree$ $at$ $most$ $n$. I am wondering whether there is other function, except polynomial, satisfies the condition of the t...

Short title.
0
Q: show kernel is valid- help please

user290331How to show that the following kernel is valid? x, y are [-1,1] K(x,y) = 1/(1-xy) Can someone help and possibly provide reasonable explanation? Also by giving any distict input x(1),...,x(m) how can we show that it is invertible?

Welcome to Math.SE, user290331. Words such as help, please do not add information to titles. Please edit the title so that it better describes the specifics of your question. Do not hesitate to make it longer or include a formula if needed. More tips here. (autocomment)Normal Human 22 secs ago
0
Q: Three urns with two types of balls problem

BrassicanI've got the following homework question: Consider 3 urns. Urn A contains 2 white and 4 black balls; urn B contains 8 white and 4 black balls; and urn C contains 1 white and 3 black balls. If 1 ball is selected from each urn, what is the probability that the ball chosen from urn A was white, ...

Title contains problem.
0
Q: determine the domain

aazz20I had a lesson about operations on funcions. Everything was good until I reach the point of division of function so the lesson was saying that you can divide a function over another function but when it comes to determining the domain i find it weird i.e f(x)=x/x+1 , g(x)=x-3/x+4 the final functi...

Short title. Question contains please.
0
Q: On the limit of a real sequence

alex alexeqThis is probably duplicated, please let me know if it is. What is the limit of the sequence $a_n = \sqrt{1 + \sqrt{2 + \sqrt{3 + \cdots + \sqrt n}} }$. Clearly the sequence is increasing and is bounded from above by 2, so it is convergent. But what is the limit?

Question contains please.
 
4:59 PM
0
Q: Questioning the wisdom of closing questions which field opinions.

Georgina DavenportIn the maths questions page I recently had a question suspended asking for support from the community on how to cope, and continue if, like me, you are doing maths because it's forced upon you as part of your course and not because you want to. Now, hopefully people will not use the opportunity...

 
0
Q: Arithmetic sequence to geometric sequence.

Amad27 The numbers $a_1, a_2, a_3, . . .$ form an arithmetic sequence with $a_1 \ne a_2$. The three numbers $a_1, a_2, a_6$ form a geometric sequence in that order. Determine all possible positive integers $k$ for which the three numbers $a_1, a_4, a_k$ also form a geometric sequence in that or...

Question contains please.
0
Q: How to multiply Roman numerals?

MariaHow to multiply Roman numerals? I need an algorithm of multiplication of numbers written in Roman numbers. Help me please.

Short question. Question contains please.
0
Q: derivative proof of $d/dx x^2$ is $2x$

zeeksI am studying for a midterm and I have no idea about how to prove that the derivate of $\frac{d}{dx}x^2$ is equal to $2x$ Anyone has any idea? Thank you

Short question. Question contains thank you. Tagged proof-verification.
1
Q: Cantor-Bernstein theorem for magmas

AlbertLet $G$, $H$ be magmas. $G_1 \subset G$ - submagma of $G$, $H_1 \subset H$ - submagma of $H$. Let $G \simeq H_1$, $H \simeq G_1$. Is true that $G \simeq H$?

Short question.
0
Q: Equations of the two lines through the origin which intersect the line $\frac{x-3}{2}=\frac{y-3}{1}=\frac{z}{1}$ at an angle of $\frac{\pi}{3}$

diyaFind the equations of the two lines through the origin which intersect the line $\frac{x-3}{2}=\frac{y-3}{1}=\frac{z}{1}$ at an angle of $\frac{\pi}{3}$. Let the direction ratios of the two required lines be $(a_1,b_1,c_1)$ and $(a_2,b_2,c_2)$. Therefore the two equations are $\frac{x-0}{a_1}...

Question contains please.
0
Q: Find the variation of A at Y

Guten TagThe functional A(Y) = 2pi*(integral 0 to 1 (absolute value(Y(x))*sqrt(1+(Y'(x))^2))dx. Find the variation of A(Y). I have no idea how to solve this problem.

0
Q: Find the Green's function for L[y] with a constant coefficient

Kimberly Ann HessI am trying to determine a $y_1$ and $y_2$ for my Green's function. I believe that I have found $y_1$, but I am a bit lost on $y_2$ because of the constant. Here is my initial problem with BC's: $L[y] = y'' + \alpha^2y = 0$, with $\alpha$ = constant; $y(0)=0$; $y(1)=0$ For $y_1$, I used $y=A cos...

Welcome to Math.SE, Kimberly Ann Hess. Questions tend to get more attention when they have a tag for a broad area of mathematics relevant to the question. Some of these tags might fit. (autocomment)Normal Human 26 secs ago
0
Q: Prove or disprove that such two graphs exist

Oria GruberProve or disprove the following statement: There are no $2$ undirected graphs $G=(V,E)$ and $H=(W,U)$ such that the following holds: 1) $G$ and $H$ are connected graphs. 2) The degrees of the vertices of $G$ and $H$ are both $1,1,2,2,2,4$ 3) $G$ and $H$ are not isomorphic. It seems to me lik...

Question contains please.
0
Q: Nth Derivative of 9* sqrt(x)

user290335Can you help me to proof that the nth derivative of $9\sqrt(x) $ is $$ (-1)^{(n-1)} *\frac{9(2n-2)!}{(n-1)!} * (4x)^{\frac{1-2n}{2}}$$ I've tried induction but didn't go very far. Many thanks

Short title.
0
Q: What is a split $\mathbb{K}$-algebra?

AnalysisStudent0414After some considerations the article I'm reading concludes: "...hence H is a simple split $\mathbb{K}$-algebra". I can't find this definition anywhere: what does "split" mean?

Short question.
0
Q: Minimum vs lowerbound

Ben BostWhat is the difference between minimum value and lower bound of a function? for me it seems that they are the same.

Short title. Short question.
0
Q: Lipschitz constant for system of ODEs

sazidConsider the system of ODEs $$u_1'=3u_1+4u_2,\\u_2'=5u_1-6u_2$$ For the system of equations above find the Lipschitz constant if we use the $\mathcal{l}_2$ norm.

Short question.
0
Q: Proof for a ring being right Artinian but not left Artinian

AaAaAaI am currently looking at the following example (and other similar examples) and I can follow the proof that it is a right Artinian ring and I also follow the example given as to why it is not a left Artinian ring. However, I do not understand what is stopping us from applying the reasoning use...

Tagged proof-explanation.
0
Q: Sum of legendre symbol

MartinIf (a,p)=1 and p is an odd prime, prove the legendre symbol sum ∑n=1 to p ((an+b)/p)=0. Where b is any integer. I Know the fact that ∑a=1 to p (a/p)=0. But I don't know how to treat with b.

Consider adding a tag for a broader subject area to which the question belongs. Some of these tags might fit. (autocomment)Normal Human 35 secs ago
0
Q: If $a^4 = 1$ and $ab = ab^2$ in a group, show that $a = 1$.

dableIf $a^4 = 1$ and $ab = ab^2$ in a group, show that $a = 1$. I haven't been able to make much progress with this question. Is there some trick that am I missing?

Short question.
0
Q: Matrix with delta function

DariusI need to prove a commutator relation, but I'm getting stuck at the definition of the matrices. $(L_{ab})_{cd} = \delta_{ac} \delta_{bd} - \delta_{ad} \delta_{bc}$ with $a<b$ and $a, b \in 1,2,3,4$. What does this definition of the matrix mean? Can someone explain this to me?

Short title.
0
Q: A factorial related inequality

ArulGiven $n$ is there an explicit or asymptotic formula for least $m$ such that $$m!\geq n?$$

Short question.
0
Q: Spline interpolation explanation

user270494 I'm trying to learn about spline interpolation, and I'm struggling to understand what h_i^3 is in the second and third equation. I don't understand how they derived that equation.

Short title.
0
Q: Is there an elementary way to see that there is only one complex manifold structure on $R^2$?

AreaManIs there an elementary way to see that there is only one complex manifold structure on $\mathbb{R}^2$? (Up to biholomorphism, naturally.) Elementary in the sense of not appealing to the uniformization theorem.

0
Q: Sum of finite number of terms of the series $\sum\limits_{L=2}^{L_{max}}\frac{1}{e^{i \phi / L} -1}$

F. JatpilGood day everyone. Are there any chances to get a compact formula for the following sum of finite number of terms? $\sum\limits_{L=2}^{N} \frac{1}{e^{ \frac{i \varphi}{L}} -1}$ N and $\varphi$ are constants. Thank you.

Title contains \limits. Question contains thank you.
Title contains elementary.
0
Q: maximum value of function with constraint

user290335Can you help me to find the maximum value of $f(x,y) = xy^2$ with the constraint that the x and y coordinates must satisfy $g(x,y) = x^2 + y^2 = 8$ How do I even start? Thanks

Short question.
0
Q: Expectation Functional in Lebesgue and Riemann Terms – Looking for a clarification

KolminHere there is a really central problem I am having self-studying probability theory, that concerns the relation between the definition of expectation in Lebesgue terms and in Riemann terms. I will truly appreciate any feedback because I feel this is really at the very core of the all theory. Sum...

Question contains thank you.
0
Q: system of differential equations in Mathematica

farrokh How can I formulate the following system of differential equations in Mathematica]1]1 for to find a general solution for $f[x_1, x_2, x_3, y_1, y_2, y_3]$? By the way I know that the solution is $f[x_1, x_2, x_3, y_1, y_2, y_3] = \frac{y_3 x_1 (y_2 x_3 + y_1 x_2 + y_1 x_3)}{(y_2 x_2 + y_3 x_3 +...

Question contains thank you. Tagged pde, differential-equations.
0
Q: How to prove that the set difference of B in A = C

worlbossCould you please help me to do this , proof . I'm currently learning set theory and am stuck on this question. Let A= Z , B ={ x ∈ Z : x= 2n + 5 for some n ∈ Z} and C = {x∈ Z , x= − 2m for some m ∈ Z} . Prove that A \ B = C.

Question contains please.
0
Q: Using Leibnitz Integral rule

realanalysis I am trying to show this using Leibnitz rule: $$D_2f(x,y) = \frac{\partial {}}{\partial{y}} \left ( \int_0^xg_1 (t,0) \ dt + \int_0^y g_2(x,s) \ ds \right)$$ $$= \int_0^x \frac{\partial{}}{\partial{y}} g_1(t,0) \ dt + \int_0^y \frac{\partial{}}{\partial{y}} g_2(x,s) \ ds$$ $$ = \int_0^x \frac{...

Short title.
0
Q: Showing that the set is disconnected

Jennie Durham I need to prove directly from the definition that $$(x,y)\in \Bbb R^2, x^2-y^2=1$$ is disconnected. Can someone please give a methodological answer? I cannot handle this type of problems Thanks in advance!

Question contains please, in advance.
0
Q: Grammer to CNF - Wierd problem

jerrymailGiven G = ({S,A,B,C,D,E}, {0,1}, P, S) where P is S --> 0A0 | 1B1 | BB | DE A --> C B --> S | A C --> S | ε D --> 0 | DE E --> EE Convert to CNF: I came up with the following but i'm stuck.... i'm not sure if I went about it correct: 1) Eliminate e-transition for C S --> 0A0 | 1B1 | BB | DE A ...

Title contains problem.
0
Q: Set of all unitary matrices - compactness and connectedness.

Sam ChristopherLet U denote the set of all nxn matrices A with complex entries such that A is unitary. Then U as a topological subspace of $C^{n^2}$ is a) compact but not connected. b) connected but not compact. c) connected and compact. d) neither connected nor compact. we can say a set is compact if it ...

Question contains please.
 
5:59 PM
0
Q: Why aren't tag scores updated in real time?

therainmakerWe all know that tag score is updated every night by a script. I think that updating tag scores seems like a relatively simple task, similar to keeping track of upvotes on answers and so on. So why is it that while upvotes and some other profile statistics are real time, tag score calculation is...

 
0
Q: Calculating a^b mod p

DiffyWhile calculating a^b mod p, i have a function f(x) in place of a. f(x)=a1*a2*a3*a4; Now i am having a hard time to understand how come a evaluates to ((a1 mod p)*(a1 mod p)*(a1 mod p)*(a1 mod p))mod p I know the basic (a*b)mod c=((a mod c)*(b mod c))mod c, but this isnt leading anywhere...

Short title.
0
Q: First passage time of the Brownian motion

CasoIn an exercise (4.1 Krapinsky, "A kinetic view of statistical physics") I am asked to show that: The probability that a brownian motion on a 1D discrete lattice never reaches the site $n$ scales as $\frac{1}{\sqrt{t}}$ in the long time limit. The brownian motion starts from the origin. My idea ...

Question contains please.
0
Q: Characteristic value of f(T) will be f(t) where t is an eigen value of T

Anurag JainLet T be a linear operator on a FDVS V over an algebraically closed field F.Let f be the polynomial over F.Prove that c is a characteristic value of f(T) iff c=f(t), where t is a characteristic value of T. Converse part is trivial. Forward part: let c is a Characteristic value of f(T) ...

Question contains please, in advance.
0
Q: Taylor series of $f(x)=\int_0^1 \frac{1-e^{-sx}}{s}ds$

mavaviljTaylor series of: $$f(x)=\int_0^1 \frac{1-e^{-sx}}{s}ds$$ at $x_0 = 0$. I've done: By fundamental theory of calculus: $$f'(x)=1-e^{-1x}$$ Which is clearly differentiable by e.g. $n$ times. What do I need to do to get the expression: $$\sum_{n=0}^{\infty}\frac{f^{(n)}(x_0)}{n!}(x-x_0)^n$$ ...

Consider adding a tag for a broader subject area to which the question belongs. Some of these tags might fit. (autocomment)Normal Human 25 secs ago
0
Q: Proof of claim that; If Im(M) is finite co-dim. then Im(M) is closed.

User2313Let $M\in \mathcal{L}(X) $, WLOG suppose $M$ is injective extend $M$ to $X\oplus Z$ s.t $M$ is surjective. Since $Im(M)$ is finite co-dim we know that $Im(M) \oplus \mathbb{R^{n}}$ is the whole range. Since we now have a bijection we get that it is a homeomophism. Furthermore, $M$ is closed since...

Tagged proof-verification.
0
Q: Easy Way to Solve $7^{99} mod (125)$

John L.Naturally, one could do this with a calculator pretty easily, but is there a trick, or something you may notice, to calculate this easily by hand? I know $\phi{125} = 100$, so could I perhaps use that somehow? I feel like I can, but I'm not exactly sure how.

Title contains easy.
0
Q: second partial derivates of a function in two variables

Giulia B.f(x,y)={(x^3y-y^3x)\over(x^2+y^2)} if (x,y)\ne(0,0) , 0 if (x,y)=(0,0). I want calculate {(\partial_(xy)f)^2 (0,0) }.

0
Q: Circles and using positions

wasifIn a basketball game a player is attempting to score. He stands at position (-10, 0). His hands are at position (-10, 15). He throws the ball and the ball follows a parabolic path that is defined as follows: y = -(x)2 + 49 The basket is at position (4, 40) and has a radius of 2. (i) Will he scor...

Short title.
0
Q: How to convert $y²=2x-x²$ into $x=f(y)$?

neuromouseMy problem is to convert the expression $y=\sqrt{2x-x²}$ into $x=f(y)$ format. I get rid of the square root, but then what?

Short question.
0
Q: How can the following integral be solved?

Razvan Paraschiv$$ \int t^3 e^{t^2+t} dt $$Solve the given integral by using the substitution techniques or the integretion by parts

Short question.
 
6:18 PM
0
Q: Merge [mllib] tag into [apache-spark-mllib]

zero323Rationale Apache Spark provides two machine learning libraries - low level spark.mllib and high level spark.ml. For questions related to spark.ml we use apache-spark-ml. Similar naming convention is used for Spark SQL (apache-spark-sql). Other Spark related libraries (GraphX, DataFrame) use shor...

 
0
Q: Simple example of a group where $g^2 = 1$ for all $g\in G$ but $G$ is not abelian.

dableI am trying to find a simple example of a group where $g^2 = 1$ for all $g\in G$ but $G$ is not abelian. All the groups I can think about with that property are abelian.

Short question.
0
Q: What are the odds of not hitting an eleven?

MikeUsing two dice what are is the probabilty of not hitting an eleven in 72 rolls?

Short question.
0
Q: Proof related to rings

Filip SulikI have a trouble proving this sentence, as I don't know what assumption should start the proving implication. I know what are the characteristics of a ring, but I m, n are not in the ring. a, b are in ring (F, +, x). m, n are whole numbers. Proof that: m x (a + b) = m x a + m x b.

Short title. Tagged proof-explanation.
0
Q: Combinatorics - Shortest route on grid with a blockade

RSparkesSuppose a grid starts at position $(0,0)$ and extends up and to the right. A shortest route along streets from $(0,0)$ to $(i,j)$ is $i+j$ units long, going $i$ blocks east and $j$ blocks north. Suppose that the block between $(k,l)$ and $(k+1,l)$ is closed, where $k<i$ and $l \le j$. How many sh...

Question contains in advance.
0
Q: Proving that the set is connected

Jennie Durham Let {$S_n$} be the collection of connected sets with the property that $S_n\cap S_{n+1} \neq \emptyset$. Prove that $\cup S_n$ are connected. Help would be appreciated! I do not know how to type $n$ goes from 1 to infinity with the sets. Thanks in advance!

Question contains in advance.
0
Q: How can the following integral be solved with substitution techniques or integration by parts?

Razvan Paraschiv$$ \int ln(2+3cos^2x)dx $$ This is considered to be a difficult integral.Unfortunately, i haven't managed to solve it by any means.

Short question.
0
Q: Probability of choosing $n$ different numbers out of $\{1,\dots,10\}$

ReveillarkConsider the set $\{1,\dots,10\}$, and suppose we draw $4$ numbers with substitution. I want to calculate the probability of drawing $n$ different numbers, with $n=0,2,3,4$. I omit the value $n=1$ because drawing $1$ different number does not make much sense. Let $\Omega=\{1,\dots,10\}^4$ be the...

Question contains in advance.
0
Q: An inequality involved probability

user115608Suppose that $(\Omega, F, p) $ is a probability space & $f $ is a measureable function that $\int _{\Omega }|f|^2 =1 , \int _ { \Omega } | f| \ge a>0$ Prove that for every $0\le \lambda \le 1$: $p (\{x\in \Omega : |f (x)| \ge \lambda a\} )\ge (1-\lambda )^2a^2$ Would anyone please give me ...

Question contains please, thank you.
0
Q: Proving a limit of a sequence based on an inequality?

Milosenter image description here I am not sure how to go with this. Thanks

Short question.
0
Q: How to prove $\sum_{k=1}^n \cos(\frac{2 \pi k}{n}) = 0$ for any n?

William ThomasI can show for any given value of n that the equation $$\sum_{k=1}^n \cos(\frac{2 \pi k}{n}) = 0$$ is true and I can see that geometrically it is true. However, I can not seem to prove it out analytically. I have spent most of my time trying induction and converting the cosine to a sum of compl...

Questions tend to get more attention when they have a tag for a broad area of mathematics relevant to the question. Some of these tags might fit. (autocomment)Normal Human 21 secs ago
0
Q: Optimization algorithm with "memory" feature

Marek GalinskiI am solving following problem: Let us have M amount of containers, where each container has it's capacity. Let us have N objects (each of certain size), that are somehow distributed into these containers. Obviously, we may not exceed the capacity of a container. I may start with some round-r...

Question contains thank you.
 
6:54 PM
0
Q: Why isn't my "bountiful" question appearing on the featured page?

ispiroI started a bounty on this question . According to What is a bounty? it's supposed to be featured on this page . It's not. Why? It's been over 15 minutes so I don't think it's caching.

 
0
Q: Algebra II Help

saraThe velocity v of a fluid flowing in a conduit is inversely proportional to the cross-sectional area of the conduit. (Assume that the volume of the flow per unit of time is held constant.) Determine the change in the velocity of water flowing from a hose when a person places a finger over the end...

Welcome to Math.SE, sara. Words such as help do not add information to titles. Please edit the title so that it better describes the specifics of your question. Do not hesitate to make it longer or include a formula if needed. More tips here. (autocomment)Normal Human 21 secs ago
0
Q: find the area of the equilateral triangle inscribed in a circle $x^2+y^2+2gx+2fy+c=0$ ?

learnerI am stuck on the following problem: How can I find the area of the equilateral triangle inscribed in a circle $x^2+y^2+2gx+2fy+c=0$ ? Answer is given to be : $\frac{3\sqrt 3}{4}(g^2+f^2-c)$ sq. unit. Can someone please explain?

Question contains please.
0
Q: Question about combining two sums

JONATHONDOEINGIf I have two sums $A = \sum_{n=0}^\infty (n+2)(n+1)a_{n+2}x_n$ and $B = \sum_{n=2}^\infty n(n-1)a_nx^n$ To combine the two sums, could I simply change the lower limit of B to 0 because it would make no difference to the value and write $$A+B = \sum_{n=0}^\infty (n+2)(n+1)a_{n+2}x_n+ n(n-1)a_nx^...

Welcome to Math.SE, JONATHONDOEING. Words such as question do not add information to titles. Please edit the title so that it better describes the specifics of your question. Do not hesitate to make it longer or include a formula if needed. More tips here. (autocomment)Normal Human 21 secs ago
0
Q: How would i visualize the set to be able to understand and answer this question

roughosingLet A be the set of all people who have ever lived. For x, y ∈ A, xRy if and only if x and y were born less than one week apart. Determine: (i) Whether or not the relation R is reflexive; I understand that x is in relation to y if x and y were born less than one week apart, but how would you me...

Title contains question.
0
Q: Prove that the line segment joining the mid-points of two sides of a triangle is parallel to the third side

aaaaaaaaProve that the line segment joining the mid-points of two sides of a triangle is parallel to the third side

Short question.
0
Q: Linear Algebra - Vector Space

Shawn EdwardI need some help with vector spaces. This is what I have: 1/3 + 5*7^1/3 + 4*7^2/3 = x + y*7^1/3 + z*7^2/3 If you rearrange to solve for 1 you get. 1 = (3+ 5x7^1/3 + 4x7^2/3)*(X + Yx7^1/3 + Zx7^2/3) Note: the "x" is multiplication. Basically I think I have to expand, I'm not sure what to d...

Short title.
0
Q: Filters and nets

TRmate True or false? a) Let $(x_{\alpha})_{\alpha\in A}$ a net over a space $X$ and $(x_{h(\beta)})_{\beta\in B}$ a subnet, where $h:B\to A$ is monotone and final. Let $\mathcal{F_1}$ be the filter generated by $(x_{\alpha})$ and $\mathcal{F_2}$ the filter generated by $(x_{h(\beta)})$. Then $\...

Short title.
0
Q: Limit of accumulation points

AndroidFishSuppose $S \subset R$ is a set and, for each $n$, $x_n$ is an accumulation point of $S$. Suppose further that $lim_{n}x_{n}=x$. Prove that $x$ is also an accumulation point of S. I was given the hints that: It is not stated that $x_n \in S$ or that $x \in S$. Approach this by the definition,...

Short title.
0
Q: summand does not exist almost everywhere

user81235Please do not provide answer here, I only needs hints and rough picture. Consider a random variable take values $\pm1$ with probability $1/2$. Prove $\lim_{n\rightarrow\infty}{S_n}$ does not exist almost everywhere.

Question contains please.
0
Q: quadratic equation and principal minors

Elie Absenter image description here to be honest I could not answer any of these questions, I'm taking this new course, but its not related to my major, can you help please, am really trying

Short question. Question contains please.
0
Q: Change of basis and diagonalisation clarification.

Kevin XuI was hoping whether someone could check if my understanding of this is correct, and also help with my question: Let us suppose that $T: V\rightarrow V$ is a linear transformation with a matrix $A$ that is similar to some diagonal matrix D, and that there is some basis of eigenvectors $e_i$. We...

Question contains thank you.
0
Q: Understanding proof using Pumping Lemma that the language {w | mid w has an equal number of 0s and 1s } is not regular

nbroI have just started reading about the Pumping Lemma, and I have some difficulties understanding the proofs. For example, in the book I am reading there's a proof for the fact that the language $$C =\{w \mid \text{ w has an equal number of 0s and 1s} \}$$ is not regular. The proof starts like ...

Tagged proof-explanation.
0
Q: Construction of Complex Numbers Inside of Set Theory

MattI am taking an introduction class on set theory. We have formally constructed the natural numbers, integers, rationals and reals. I am now trying to think of how to define the complex numbers inside of set theory. My idea is to follow the idea of how the reals were constructed from Dedekind cut...

Question contains thank you, in advance.
0
Q: uniformly continuous (lnx^2)/(1-x^2) on (-1,1)

Wiktoria CzerwonkaIs f(x)= (ln(x^2))/(1-x^2) uniformly continuous on (-1,1)? I know the definition of uniform continuity. and when the derivative is bounded on (-1,1) f is UC.

Short question.
0
Q: notation for substitution in lambda calculus

Marcus Junius BrutusI get the substitution notation in lambda calculus for "simple" applications such as: (λx.x+1)(5)=[5/x](x+1)=5+1=6 What I don't get is how that works when I pass a lambda as the "parameter". E.g. to compose functions f(x)=x+1 and f(y)=2*y: (λy.2*y)(λx.x+1)=[λx.x+1/y](2*y)=2*(λx.x+1) (??) Th...

Consider adding a tag for a broader subject area to which the question belongs. Some of these tags might fit. (autocomment)Normal Human 21 secs ago
0
Q: Densly embedded ?confused

openheimerLet $H$ be a hilbert space and $V$ is a banach space. What doese it mean $V$ is densly embedded in $H$ is it necessary that $V\subset H$

Short title. Title contains confus. Short question.
0
Q: Density results

ZakI have a Reflexive Banach space V that is compactly embedded into a Hilbert sapce H which is continuously embedded in its turn into a Banach space W:= V' (with W being the dual of V, that is V') and with H being identified with its dual. How can I prove that V is dense in H? is it possible for So...

0
Q: How to generalize this code in MATLAB

user162343this time I am asking for help to generalize the code for the QR-factorization, I did the following code for the fist step: %Script for the Householder QR factorization. A=[60,41,-88;42,60,51;0,-28,56;126,82,-71]; disp(A) m=length(A(:,1)); n=length(A(1,:)); I=eye(m); e1=I(:,1)'; %We compute th...

Question contains please, in advance.
0
Q: which option should I pursue for study away for a semester?

HumbleStudentI am an international student currently studying in the US, pursuing a B.A. in mathematics at a relatively unknown Liberal Arts College. My school is small and does not have many course options as far as mathematics courses are concerned, because of its liberal arts nature. However, I was awarded...

Tag (soft-question) should not be the only tag a question has. Please add a tag for a subject area to which the question belongs. (autocomment)Normal Human 20 secs ago
0
Q: When is the function $f=ae^{-x}$ in the open unit ball $(X,d_u)$?

Al jabraWhere $d_u$ is the uniform metric and $f=ae^{-x}$ I know the solution involves the step $d_u (f,0)=sup |ae^{-x}|=|a|$ Why does this equal $|a|$?

Short question.
0
Q: Show that if $S$ is an orthogonal set of nonzero vectors in $\mathbb{R}^n$ then $S$ is a linearly independent subset of $\mathbb{R}^n$.

Alex Use the dot product to show that if $S = \{\vec{v_1}, \vec{v_2}, ..., > \vec{v_n}\}$ is an orthogonal set of nonzero vectors in $\mathbb{R}^n$ then $S$ is a linearly independent subset of $\mathbb{R}^n$. I believe that since the vectors are orthogonal to one another, this implies they are ...

Tagged proof-verification.
0
Q: Prove that $\{w \mid \text{ w has even length and the first half of w has more 0s than the second half of w} \}$ is not regular?

nbroI have had some difficulties understanding proofs that a language is not regular using the Pumping Lemma, and now I need to prove that the following language $$A = \{w \mid \text{ w has even length and the first half of w has more 0s than the second half of w} \}$$ is not regular. I would sta...

Question contains please. Tagged proof-writing.
0
Q: ln(x^2+y^2)/(1-x^2-y^2) uniformly continuous on (x,y):x^2+y^2<1?

Wiktoria CzerwonkaIs ln(x^2+y^2)/(1-x^2-y^2) uniformly continuous on (x,y):x^2+y^2<1? Please help.

0
Q: Find $a$ such that $u_n = \dfrac{a^{2n} + a^{n+1} + 3a - 5}{a-1}$ is not a perfect square for only finitely many $n$

primitiverootLet $a$ be a positive integer greater than 1 and define the sequence $u_n$: $$u_n = \dfrac{a^{2n} + a^{n+1} + 3a - 5}{a-1}$$ Find all possible value of $a$ such that $u_n$ is not a perfect square for only finitely many values of $n$ After a few attempt I found out the result is $a \in \{2,10\}$ ...

0
Q: Prove that minimum spanning tree is a tree

Todd SewellFrom the the Wikipedia page Minimum spanning tree: A minimum spanning tree is a spanning tree of a connected, undirected graph. It connects all the vertices together with the minimal total weighting for its edges. Let G be a connected, undirected graph, and let H be a connected subgraph of ...

Tagged proof-writing.
Tall formulas in titles break the layout of question lists. Please replace \dfrac with \frac in the title. (autocomment)Normal Human 21 secs ago
0
Q: Determinant tridiagonal matrix

Roos JansenCan anybody help me out with getting an expression of the values of $\lambda$ for a matrix $A$ for which $det(A-\lambda I)$ equals the determinant of a matrix with on the main diagonal $\lambda$, on the diagonal above the main diagonal $\dfrac{1}{2}$ and on the diagonal under the main diagonal $\...

Short title.
0
Q: counterexample showing that Maschke's Theorem does not hold if characteristic divides group order

user80703I am taking a graduate Algebra course, and we were given the following example to see that Maschke's Theorem does not hold if the characteristic of the field F does divide the order of G: Let $F = \mathbb{F}_2$, $G = \mathbb{Z}/\mathbb{Z}_2$ and $V=FG$. Then show that $W = span\{0+1\}$ is not co...

Question contains thank you.
0
Q: if G acts trivially on A then what can we say about $H_1(G,A)$

aliif G acts trivially on A then $H^1(G,A)=Hom(G,A)$.if $A=\mathbb{Z}$ then $H_1(G,\mathbb{Z})=G/G'=G/G'\otimes \mathbb{Z}$ can we say $H_1(G,A)=G/G'\otimes A$?(i think it is natural to define $G\otimes A=G/G'\otimes A$ so we have $H_1(G,A)=G\otimes A$

Questions tend to get more attention when they have a tag for a broad area of mathematics relevant to the question. Some of these tags might fit. (autocomment)Normal Human 21 secs ago
0
Q: Principal directions

Ana HiaWe define the Dupin indicatrix to be the conic in $T_PM$ defined by equation $II_P(V,V)=1$. If P is a hyperbolic point, show that the principal directions are the symmetry axes of the indicatrix When P is a hyperbolic point, the indicatrix is a hyperbola with equation $k_1*x²+k_2*y²=\pm1$ I do...

Short title.
0
Q: Upper bound for chromatic number related to number of edges

Diana CremarencoProve that in any simple, undirected graph $G=(V,E)$, the chromatic number $χ(G)≤ \sqrt{2m} +1$, where $m=|E|$

Short question.
 
8:09 PM
-1
Q: Is my audit completly wrong about this "title only" question?

MaloubobolaI failed earlier today an audit on this topic. I flagged it as very low quality because there is just a title. For me the problem is (quote from the flag description) This question has severe formatting or content problems. ... But it seems this is a hight quality question (Now I can see...

0
Q: "loggin via Google +" button fails if e-mail is entered

ShawnRepro: Log out of stackoverflow Click on "Ask a question" Login screen appears Enter your e-mail Click the Google+ button above EXPECT it to log you in via Google+ OBSERVE it complain about a wrong password / could not find an account for that email address This is a weird flow of cours...

 
0
Q: diophanic equations for large numbers

user1361268I am trying to find number of positive solutions for linear diophantine equation ax + by = c. I followed this answer http://math.stackexchange.com/a/80886/290384 ...there is written that the solution is: floor(c/(ab)) or floor (c/(ab)) + 1 I can't decide when to add the one to the result. For e...

Question contains thank you.
 
0
Q: How is the review ban time calulated?

llanatoI have been reviewing as much as regularly as possible, I've been getting all the test questions correct and I got one wrong just there and it's saying that I can't review for another 7 days as I got so many wrong, how can a 7 day ban be justified for getting one single test question incorrect?!

 
0
Q: Why relation divisibility is not relation partially ordered set on set Integer?

user288083I try get it why relation divisibility is not relation partially ordered set. $A=\{−2, 2, 4, 6, 8, 10\}$ with relation divisibility "|" $R$ is relation divisibility | when $a,b,c \in Z : a = b \cdot c$ For relation partially ordered set must be relation: reflexive (fulfil) - everery number ...

Question contains please.
0
Q: Formulate the dual problem for primal problem with absolute value constraint

Fence JumperLet $y \in R$, the goal is to find the dual problem to: $$\min y\\ s.t. |y| \leq 0$$ The lagrangian of the problem is: $$L(y, \lambda) = y + \lambda|y|$$ The dual function is: $$g(\lambda) = \inf_y y + \lambda |y|, \lambda \geq 0$$ But the constraint $|y| \leq 0$ is only satisfies at $y = ...

Title contains problem, problem.
0
Q: Connected and disconnected sets

Jennie Durham If A and B are connected, then does it imply that their union is connected? For example, if $A=[0,2], B=[4,5]$. Is the union connected? I believe the intersection is not necessarily connected, but how about this example? Thanks in advance!

Question contains in advance.
0
Q: Values of z for which a recursive sequence converges or diverges.

user131715I am currently working on a problem and I at a loss as to where to begin. Consider a recursive sequence x_n, where: x_0 = 0 x_n = z^(x_n-1) I need to find the values of z for which the sequence is convergent. I have currently plugged in some sample values of z into this sequence and this is w...

This site uses MathJax formatting of formulas. More tips here. (autocomment)Normal Human 21 secs ago
0
Q: Variation and derivative

SaladinosWhen in general is true that $\nabla_{\mu}\delta=\delta\nabla_{\mu}$ where $\nabla_{\mu}$ is covariant derivative?

Short title. Short question.
0
Q: Derive a power series expansion about x=0

JONATHONDOEING$(1+x^2)y''(x) -xy'(x) + y(x) = 0$ My attempt so far: I know we are looking for an ordinary point because $a(x) =\frac{x}{(1+x^2)}\ $ and $b(x) = \frac{1}{(1+x^2)} $ are both analytic. Also I know y = $\sum_0^{\infty}a_nx^n$ The way I have been taught to compute this sum is to differentiate the...

Title contains 0.
0
Q: Prove that $\lambda = 0$ is an eigenvalue if and only if A is singular, without using $\lambda_1\cdot\ldots\cdot\lambda_n = det(A)$.

Victor PintoI would like to know if there is any proof without using the fact that: $$\lambda_1\cdot\lambda_2\cdot\ldots\cdot\lambda_{n-1}\cdot\lambda_n = det(A)$$ I managed to prove that if $\lambda = 0$ then, A is singular, by: $$ det(A-\lambda\cdot I)=0 \Rightarrow det(A-0\cdot I)=0 \Leftrightarrow det...

Tagged proof-writing.
0
Q: Proving almost sure convergence

David SouthAssume the sequence of random variables $X_1, X_2, \cdots$ are IID with finite mean and finite variance. Define a random variable: \begin{align} Y_n = \frac{X_n}{n} \end{align} Show that $Y_n \to 0$ almost surely. To converge to some value almost surely implies: \begin{align} \mathbb{P}\left( \...

Question contains thank you.
 
9:02 PM
0
Q: Show $g=3$ is contained in the unit ball of $X=C[0,1]$ in the uniform metric.

Al jabraHow do I show that the function $g=3$ is contained in the unit ball of $X=C[0,1]$ in the uniform metric? I know the uniform metric $$d_u(g,0)=sup |3|$$ where do I go after this?

Short question.
 
0
Q: How does this site differ from Theoretical Computer Science Stack Exchange?

JesseTGQuestion's in the title; what makes Computer Science Stack Exchange differ from Theoretical Computer Science Stack Exchange?

0
Q: Victim of serial unervoting?

harrymcVery unusual - within 4 minutes I got 3 undervotes : Open file with emacs as administrator on windows Outlook refuses to send email with non-matching address in cert Using a firewall to discriminate items in “Host process for windows services” that abuse metered data (The last one was candidate...

 
0
Q: Does this norm inequality hold?

user143763$\int_0^te^{A(t-\tau)}Bu(\tau)d\tau$ is rewritten as $(e^{\cdot A}*Bu)(t)$ by using convolution product, where $e$ is the natural logarithm base, $A\in\mathbb R^{n\times n}$ and $B\in\mathbb R^{n\times m}$ are constants, $u_1(t)$ and $u_2(t)\in L^\infty([0,T],\mathbb R^m)$. Then for the convoluti...

Question contains in advance.
Okay, enough of "thank you in advance" reports here.
0
Q: Showing that 3 dimensional unit ball is connected

Jennie Durham Let $$(x,y,z)\in \Bbb R^3, x^2+y^2+z^2=1$$ I need to show that this set is connected. Help would be appreciated! Thanks in advance!

Short question.
0
Q: Transform cos to e function

hr0mWhat are the steps in order to transform the cosine function to the exponential function: $$ [\cos(k \pi/N)]^2 \Rightarrow e^{-n/2 (k \pi /N)^2} $$ if i know that $$ 0 < k/N << 1 $$ I just want to understand this step. I am awere of the Euler's formula, yet i can't figure out, how to use it. T...

Short title.
0
Q: Solving complicated trigonometric limits

StefanCan someone help me solve this limits? I know how to solve basic trigonometric limits, but this is too advanced for me. I know that i should start by writing the tgx=sinx/cosx. Some instructions after that?

Consider replacing (analysis) with a more specific tag for the relevant branch of analysis. (autocomment)Normal Human 21 secs ago
0
Q: I need to show the an expectation is finite

MandiLet X and Y be nonnegative random variables with $EX^4 + EY^4 < \infty$, Show that for any $r,s \in (0, \infty)$, with $r+s=4$, $EX^rY^s <\infty$

Short question.
0
Q: Prove that there are 4 differents positive integers (a,b,c and d) that satisfy both conditions: "a² + b² = c² + d²" and "a+b=c+d"

rodrigorandelI've found a similar question here but what I'm looking for is a little bit different... Besides trying to prove that both these conditions are satisfied and a,b,c and d are four different positive integers: a² + b² = c² + d² a + b = c + d I would like to prove that this holds for n t...

Welcome to Math.SE, rodrigorandel. This site uses MathJax formatting of formulas. More tips here. (autocomment)Normal Human 21 secs ago
0
Q: Category theory without sets

mrpI have been reading Mac Lane's Categories for the Working Mathematician, and the prospect of developing category theory without any use of set theory is mentioned more than once in the book, but never actually realised. I was wondering whether there are any good references (books or online notes)...

Short title.
0
Q: Help solving inequality

1westIn an exercise question, I am asked to show that: Given $q>p>3$, where p and q are distinct odd prime numbers. Show that $p > 3 + \frac{6}{q-3}$ and conclude that $\phi(n) < n < \frac{3\phi(n)}{2}$. To prove the first part of this I did the following: Proof by Contradiction: Suppose tha...

Words such as help do not add information to titles. Please edit the title so that it better describes the specifics of your question. Do not hesitate to make it longer or include a formula if needed. More tips here. (autocomment)Normal Human 21 secs ago
0
Q: Money Business Problem

Omari Horne JoynerDan's DVD discount dungeon is going out of business and selling al of its DVDs. The first customer bought 1/6 of all the dvds plus 1; the second buyer again took 1/6 of the remaining DVDs plus 2; the third buyer bought 1/6 of the remaining DVDs plus 3, and so on. When Dan had sold all of the DVDs...

Short title. Title contains problem.
0
Q: How to get rid of symbol manipulation in math?

fahad aijazI have recently started to develop my mathematical intuition. In the past I saw math as a mere game of symbol manipulation, whosoever was able to see patterns and cram formulas and apply them upon those symbols won. However,I now feel that the truth is contrary to the aforementioned,this is becau...

Question contains please.
0
Q: $ (e^{2x})=(e^x)^2$ Can Some one explain how these two are the same? I know it is trick, but I cannot see how it fits into the exponent rules.

BenCarson2016$ (e^{2x})=(e^x)^2$ Can Some one explain how these two are the same? I know it is trick, but I cannot see how it fits into the exponent rules.

Short question.
0
Q: Element over Q with Minimum polynomial

Markenter image description here Guys I am so lost on this question. Can someone please help me with this.

Short question. Question contains please.
0
Q: Weak convergence function

matt smithI am studying for an exam and this was one of the hard problems in my textbook. Let $f_n(x) = 1 − cos(2πnx)$ for n $\in$ N and x $\in$ [0,1]. Verify that $f_n$ is the density of a probability measure $μ_n$ on [0, 1]. Is there a weak limit $μ_n \rightarrow^w$ μ? Either show convergence and ide...

Short title.
0
Q: How to find the limit of cos(2x)/x as x approaches zero

Earl SmithBasicly the title. I know it's easy but I can't figure it out

Short question.
0
Q: What is the intuitive difference between $E(X 1_A)$ and $E(X|A)$?

user262959I know that they are not the same since $E(X|A) = \frac{E(X 1_A)}{Pr(A)} $ (*) But without using formula (*) above, for me, both of them means "Average of X on an event A". Could you please explain for me intuitively what is the subtle difference between $E(X 1_A)$ and $E(X|A)$? Thank you.

Questions tend to get more attention when they have a tag for a broad area of mathematics relevant to the question. Some of these tags might fit. (autocomment)Normal Human 21 secs ago
0
Q: Show that there exists a mixed strategy for player 2 that strictly dominates his strategy B.

user290427 Player 2 A B C Player 1 x 6,10 0,2 3,0 y 0,0 1,4 4,6 I tried removing B from the equation as it is supposed to have been strictly dominated but it doesn't give me the right answer, i dont know how to go about answering this.

Welcome to Math.SE, user290427. Consider adding a tag for a broader subject area to which the question belongs. Some of these tags might fit. (autocomment)Normal Human 21 secs ago
0
Q: Does the Poisson bivector give rise to an integrable distribution?

Daniel Robert-NicoudI am reading the book Lectures on the geometry of Poisson manifolds, by Izu Vaisman. To a Poisson structure $\{\cdot,\cdot\}$ on a manifold $M$ we associate the Poisson bivector field $w\in\Gamma(\Lambda^2TM)$. We have Lemma: $[w,w]=0$ This follows directly from proposition 1.4 in the book...

Consider adding a tag for a broader subject area to which the question belongs. Some of these tags might fit. (autocomment)Normal Human 21 secs ago
0
Q: If the left regular representation of a finite group G contains an odd permutation, then G has a subgroup of index 2

Henri LI got to the point where I showed: $$[\pi(G):A_G \cap \pi(G)]=2$$ I have a trouble convincing myself of the final step. So if $\pi:G \rightarrow S_G$ is faithful (i.e. $\pi$ is injective), then $G$ can be identified with $\pi(G)$ and thus the preimage of $A_G \cap \pi(G)$ gives the subgroup of i...

Title contains 2. Tagged proof-verification.
0
Q: Random selection by coin toss

EuclydeI have 81 episodes of Midsomer Murders. I have watched them all, so I like to pick one "at random" to watch now. Can I randomly select one by dividing them in half over and over, flipping a coin each time to pick which half to keep until I get down to one? If I get an odd number at any stage, bec...

Short title.
0
Q: Differential of definite integral

HesamWhat is the solution of $$ \frac{d}{dy} \int_{0}^{y} exp(-y(x+1)) dx $$ If there was no y inside exponential function the answer was the exponential function.

Short question.
 
9:59 PM
2
Q: Error text is hard to read on team page

bjb568 Uh… how many seconds is that? The background should be red, not light grey.

 
0
Q: Functions differentiable at zero

SelfStudyIs there a case where a function $f$ that is not differentiable at $0$ and a function $g$ that is differentiable at $0$ where $f+g$ is differentiable at $0$.

Short question.
0
Q: Prove or disprove this equivalence relation

bellaLet R be a relation defined on the set N by a R b if either a|2b or b|2a. Prove or disprove: R is an equivalence relation.

Short question.
0
Q: Finding all group homomorphisms $(\mathbb{Q},+)\to (\mathbb{Q}-\lbrace 0\rbrace,\cdot)$

capIf $f$ is such a homomorphism, then $f(x+y)=f(x)f(y)$. I know of examples of $f$ which satisfy this property, such as $f(x)=e^x$, but how do I find all of them?

Short question.
0
Q: factoring cubic polynomail when answer involves decimals

starbI am trying and not understanding how to factor. I have tried long division and synthetic division but am not able to factor -x^3+7x^2-10

Short question.
0
Q: Question regarding career in Mathematics

user290425I apologize if this is not the correct place to ask such a question. This is my first time using Math StackExchange. Please feel free to point me to another site if this is not the correct place to ask this :) I just completed my undergraduate degree in Mathematics and Applied Mathematics. The r...

Welcome to Math.SE, user290425. Words such as question do not add information to titles. Please edit the title so that it better describes the specifics of your question. Do not hesitate to make it longer or include a formula if needed. More tips here. (autocomment)Normal Human 21 secs ago
0
Q: Averaging Problem from Stein and Shakarchi

AthertonAttached is an image which i don't quite understand: Why can he pull the value $f(x)$ inside the integral when $f$ is continuous? Then i don't see where he uses the ball $B$ being of radius less than $\delta/2$

Title contains problem.
0
Q: Factor polynomial

JuhoHow to factor following polynomial into its lowest terms? $$x_i^n+(n^n-1)\prod _{i=1}^nx_i.$$ That question comes from inequality $$(\sum_{i=1}^n x_i)^n \ge x_i^n+(n^n-1)\prod _{i=1}^nx_i,$$ where $x_1,x_2,\ldots,x_n>0$.

Short title.
0
Q: Does the following serie converge or diverge?

user290431Does ∑ln(n)/(n*n^(1/3)) converges or diverges? Which test should i use? i tried the ratio test and root test but both of them are inconclusive so tried it by comparison test but i don't know which function i have to compare with. Thanks!! ∑ln(n)/n^(4/3)

Welcome to Math.SE, user290431. This site uses MathJax formatting of formulas. More tips here. (autocomment)Normal Human 21 secs ago
 
10:17 PM
-1
Q: Should we move [sharepoint] to sharepoint.se

thesecretmasterI noticed that we have a lot of questions tagged sharepoint, and also a sharepoint SE site. Would we want to move those questions? Can we?

 
0
Q: matrix inverse question and AB=BA

Brian JonesSo, matrix A * its inverse gives you the identity matrix correct? Also, if you have AB=BA, what does that tell you about the matrices?

0
Q: Limits of finite sums

Gustavo VianaI know that: $$\int_0^1 1 - x^2 dx = \frac{2}{3}$$ And that represents the area below the curve, delimited by the lines $x= 0$ and $x = 1$ But $$\int_0^1 1 - x^2 dx = \lim_{n \to \infty} \left( 1 - \frac{2n^3 + 3n^2 +n}{6n^2} \right) $$ And this limit is obtained by calculating an approximation ...

Short title.
Words such as question do not add information to titles. Please edit the title so that it better describes the specifics of your question. Do not hesitate to make it longer or include a formula if needed. More tips here. (autocomment)Normal Human 21 secs ago
0
Q: Diffentiable function only at zero

SelfStudyIs there a case where $f$ is differentiable at $0$ but not any other point?

Please don't use (self-learning) tag just because you were self-studying. This tag is only for questions about the process of self-studying. (autocomment)Normal Human 21 secs ago
0
Q: Sketch $\begin{cases}\dot x =x-y \newline\dot y= x+y\end{cases}$ on a phase portrait.

Meitar Sketch $\begin{cases}\dot x =x-y \newline\dot y= x+y\end{cases}$ on a phase portrait. The original question requires me to change the variables to polar ones, but then asks me to draw a phase portrait of the problem using the prior form and then the new form, and I am currently stuck with drawing...

Title contains \begin.
0
Q: A short question on how the following statement is induced: Groebner Basis Lemma

Melba1993I have a very short proof here for the following lemma, and there's one small bit I am not sure why it is true. Lemma: Let $G$ be a Groebner basis for the polynomial ideal $I$. Let $p\in G$ be a polynoial such that $LT(p) \in <LT(G-\{p\})>$. Then $G-\{p\}$ is also a Groebner basis. The foll...

Title contains question. Question contains please.
0
Q: Decomposing a set of complex matrices into orbits of the operation of conjugation

yung_PabsI need some assistance with the proof for part (b) of the following problem statement: Problem Statement: Decompose the set $\mathbb{C}^{2\times2}$ of $2\times2$ complex matrices into orbits for the following operations of $GL_{2}(\mathbb{C})$: (a) left multiplication (b) conjugatio...

Tagged proof-explanation.
0
Q: Do monotonic transformation of supermartingales preserve martingale properties?

Daniel Wills$x_t$ is a super-matingale. And f is an increasing function. Can does $f(x_t)$ has any martingale property? What is f is decreasing?

Short question.
0
Q: orbits of matrices in GL2

user289859Is there a formula for calculating the cardinalities of orbits of $A\in \text{GL}_2(F)$ under conjugation? I know that $|G|=|G_x||Gx|$.

Short title. Short question.
 
10:56 PM
1
Q: The search fails on some wildcard-tag-searches

DeduplicatorTrying to the search for [*-s] is:q fails to yield any result, despite the tag strcat-s having seven questions. If one leaves off the operator (searching for [*-s]), the search implodes and redirects to the all-questions-page.

 
0
Q: Hausdorff dimension of homeomorphic compact metric spaces

sva1) Are there examples of homeomorphic compact metric spaces of different Hausdorff dimension? 2) If yes, are there sufficient conditions on the spaces which would imply the equality of Hausdorff dimensions? In my case the spaces are the so called multiple conic singularities (MCS-) spaces. The...

Consider adding a tag for a broader subject area to which the question belongs. Some of these tags might fit. (autocomment)Normal Human 21 secs ago
0
Q: Improper integrals.

yasir$ I=\int _{ -\infty}^{ \infty} \frac{log(\lvert t\rvert)}{x^4+t^2}dt $ ATTEMPT:- Since the function is even, the following property may be used: $\int _{-a}^a f(x)dx=2\int_0^a f(x)dx$, if $f(x)=f(-x)$$ \qquad$$-(1).$ $\implies I=2\int _{ 0}^{ \infty} \frac{log(\lvert t\rvert)}{x^4+t^2}dt $ H...

Short title.
0
Q: Projection in n-ary relation.

AashiSo I got this question in my exam and I couldn't solve it. Later my professor gave me the solution but I'm not getting it properly. I guess my concepts on projection are not that strong. Can you please help me understand it? Question: Give an example to show that if R and S are both n-ary relatio...

Short title. Question contains please.
Short question.
0
Q: Please,give my answer.

Nader RihanHow can I relate any two functions to each other.

0
Q: Help us disprove a self-conjured statement of almost sure convergence.

David SouthDefine a sequence of RVs $(X_n)$ that only take on integer values. We want to show it does not converge to some integer "$l$" almost surely. We prove by contradiction: Assume $(X_n) \to l$ almost surely. Then, $\exists N$ such that $n > N \implies X_n = l$. This would force $P(X_n = l) =1$ $\fo...

Title contains help.
Welcome to Math.SE, Nader Rihan. Words such as please do not add information to titles. Please edit the title so that it better describes the specifics of your question. Do not hesitate to make it longer or include a formula if needed. More tips here. (autocomment)Normal Human 29 secs ago
0
Q: Motion of a particle.

WAS I first started by integrating both sides with respect to t (dt). It says that B is along the z-axis but how do I account for that.

Short title. Short question.
0
Q: Temple of Horus: area of a quadrilateral formula

KatrinaThe Temple of Horus at Edfu, Egypt, has a formula for finding the area of an arbitrary quadrilateral with sides a,b,c,d as (a+c)/2 * (b+d)/2. First, show that this formula is correct if the quadrilateral is a rectangle, then show that the formula gives too large an answer for any other quadrilat...

Welcome to Math.SE, Katrina. This site uses MathJax formatting of formulas. More tips here. (autocomment)Normal Human 21 secs ago
0
Q: Calucus Problem I don't understand and need help answering

user290452The curve whose equation is y = ax^3 + bx^2 + cx +d has a point of inflexion at (-1,4), has a turning point wen x=2 and passes through the point (3,-7). Find the values of a,b,c,d and the position of the other turning point

Welcome to Math.SE, user290452. This site uses MathJax formatting of formulas. More tips here. (autocomment)Normal Human 21 secs ago
0
Q: Is there a simple solution for $ \frac{d}{dy} log( \int_{0}^{y} \exp(-y(x+1))\, dx ) $

HesamWhat is the solution of $$ \frac{d}{dy} log( \int_{0}^{y} \exp(-y(x+1))\, dx ) $$ The solution without log was explained here.

Short question.
0
Q: Baby Rudin Exercise 10.24

SokeThe exercise: Let $\omega = \sum a_i(\mathbf x) dx_i$ be a 1-form of class $\mathscr{C}''$ in a convex open set $E \subset \mathbb{R}^n$. Assume $d \omega = 0$ and show that $\omega$ is exact in $E$, by completing the following outline: Fix $p \in E$, Define $$f(\mathbf{x}) = \int_{\bf [p,x]} ...

Short title. Title contains exercise.
0
Q: Prove (without using derivatives) that a function is strictly increasing.

klorzanAs an exercise, we want to prove that the following function is strictly increasing, without using derivatives. $f(x)=x^3-\frac{1}{2} x$ Using derivatives, this is a very simple proof, but since we are not able to, my method approach begins with looking at the definition of strictly increasi...

Tagged proof-writing.
0
Q: If $\sum_{n=0}^\infty a is a divergent series, does that mean $\sum_{n=0}^\infty sin(a) is also divergent?

kimLeeGiven a divergent series a sub n, is the sine of that series also divergent? If not, give a counterexample.

Short question.
0
Q: Showing that a set is countably infinite by defining a bijection between N and that set.

Hani Al-shafeiI'm a little confused on what is being asked here: Show that the following sets are countably infinite, by defining a bijection between N (or Z+) and that set. -The set of positive integers divisible by 5. -{1,2,3} X Z.

Tagged proof-writing.
0
Q: derivation of backward probabilities $\beta$ Hidden Markov Model (message passing). Any help in completing it?

fstabI am trying to formulate in a recursive manner the backwards probabilities $\beta$ of a Hidden Markov Model where $w_i$ are the observed symbols and $s_i$ are the latent states. Is the following derivation correct? Do you know if there is any way without reformulating everything from scratch, to...

Title contains help.
0
Q: How to write this constraint?

5.r.aI have an integer variable $q_i$ and a binary variable $x_{ ij }$. How can I write the following constraint? if $x_{ ij } = 0$ then $q_i = 0$ else if $x_{ ij } = 1$ then $q_i > 0$

Short title.
0
Q: $b_n \ge 0$ and the series of $b_n$ converges. $a_n \ge 0$ and $lim \frac{a_n}{b_n} exists$.

evan318$b_n \ge 0$ and the series of $b_n$ converges. $a_n \ge 0$ and $lim \frac{a_n}{b_n} exists$. Does the series of $a_n$ converge?

Short question.
0
Q: A tricky question using the Second Derivative Test to show that a point is a local minimizer,

user290432Suppose we are given a function $f(x)$ with two continuous derivatives which satisfies the differential equation $xf′′(x)+3x(f′(x))^2=1−e^{−x}$ for all real x. (Do not attempt to solve this differential equation.) Part 1 If f has an extremum at a point c, show that this extremum is a (local) ...

Title contains question, tricky.
 
11:39 PM
0
Q: What to do if a question has been flagged as a "possible duplicate" but isn't a duplicate

TMinWhat to do if a question has been flagged as a "possible duplicate" but isn't a duplicate? This question is marked as a possible duplicate, but it is not a duplicate (at least not of the question that is linked to in the flag). The first question is asking how to convert an int into an NSIntege...

0
Q: Appropriate response

wayfarerWhat's the appropriate response to this (at "Programmers") "You are asking a fairly wide open question and then asking opinions on possible answers. It sounds like you haven't really done your home work. Please show your research." I responded with the below and then deleted the question, beca...

 
11:57 PM
0
Q: Volume of a parabolaid and the plane

Beshoy AwadCan someone help me start is problem off. Find the volume bounded by the parabolaid x^2 + y^2 + z =7 and the plane z=1. Thank you

Short question.
 
00:00 - 16:0016:00 - 00:00

« first day (13 days earlier)      last day (561 days later) »